100% satisfaction guarantee Immediately available after payment Both online and in PDF No strings attached
logo-home
LSAT Logical Reasoning Question Types - Top Down or Bottom Up || Questions and 100% Verified Answers. $11.33   Add to cart

Exam (elaborations)

LSAT Logical Reasoning Question Types - Top Down or Bottom Up || Questions and 100% Verified Answers.

 11 views  0 purchase
  • Course
  • LSAT Logical Reasoning
  • Institution
  • LSAT Logical Reasoning

Which of the following, if true, most helps to resolve the apparent conflict in the statement above? correct answers Paradox: "helps to explain" a situation or "resolve the apparent conflict" - what are the two things at issue/ conflicting in the stimulus? - the correct answer will allow for ...

[Show more]

Preview 2 out of 9  pages

  • August 9, 2024
  • 9
  • 2024/2025
  • Exam (elaborations)
  • Questions & answers
  • LSAT Logical Reasoning
  • LSAT Logical Reasoning
avatar-seller
FullyFocus
LSAT Logical Reasoning Question Types - Top Down or
Bottom Up || Questions and 100% Verified Answers.
Which of the following, if true, most helps to resolve the apparent conflict in the statement
above? correct answers Paradox: "helps to explain" a situation or "resolve the apparent conflict"

- what are the two things at issue/ conflicting in the stimulus?

- the correct answer will allow for BOTH of those things to be true at the same time = SOLVE
THE MYSTERY!!

(BOTTOM UP = STRONG/ SPICY!!)

The reasoning in the argument is flawed because the argument correct answers Flaw.

Ask yourself:
1. What is the argument failing to consider?
2. What is it assuming?

Answer Choices:
1. Did it happen?
2. Is it a problem?

(TOP DOWN = MILD)

which of the following most accurately expresses the overall conclusion drawn in the argument?
correct answers Main Point (top down)

Which of the following principles, if valid, most helps to justify the columnist's judgment that
the legislator's proposal is unacceptable? correct answers Strengthen/ Principle.

Because the principle (premise) will be listed in the answer choices, this is a strengthen question
because the correct answer will help "justify the columnist's judgment."

(BOTTOM UP = STRONG/SPICY)

Which of the following is an assumption required by the argument? correct answers Necessary
Assumption: DEPENDS, REQUIRES or RELIES
- (TOP DOWN = MILD)

Which of the following most accurately expresses the conclusion drawn in the citizen's
argument? correct answers Main Point. "conclusion drawn" = Main conclusion (TOP DOWN =
MILD)

, The dialogue provides the most support for the claim that Peraski and Jackson disagree over
whether correct answers Disagree (TOP DOWN = MILD)

Which of the following, if true, most helps to resolve the apparent discrepancy in the information
above? correct answers Paradox "resolve the apparent discrepancy"

- what are the two things at issue/ conflicting in the stimulus?

- the correct answer will allow for BOTH of those things to be true at the same time

(BOTTOM UP = STRONG/ SPICY!!)

The conclusion of the argument can be properly drawn if which one of the following is assumed?
correct answers Sufficient Assumption.

The question asks for something that, IF ASSUMED, would allow the conclusion to be ROCK
SOLID!

Why does the support not guarantee the conclusion? What is the gap?

** like a super strengthen question

(BOTTOM UP = STRONG/ SPICY!!)

Of the following claims, which one can most justifiably be rejected on the basis of the statement
above? correct answers "Most likely False"

- similar to as most strongly supported question, but what cannot be true based on the statements
above? The choices that could be true or must be true can be rejected.

(TOP DOWN = MILD)

Which of the following, if true, most strengthens the argument? correct answers Strengthen:

Ways to Strengthen:
1. Deny alternative explanation
2. Promote evidences relevance to conclusion
3. Add support for/ validate a study
4. Add additional relevant info

(BOTTOM UP = STRONG/ SPICY!!)

Which of the following, if true, most helps to explain the failure of the strategy involving
wolves? correct answers Paradox: "helps to explain" a situation

- what are the two things at issue/ conflicting in the stimulus?

The benefits of buying summaries with Stuvia:

Guaranteed quality through customer reviews

Guaranteed quality through customer reviews

Stuvia customers have reviewed more than 700,000 summaries. This how you know that you are buying the best documents.

Quick and easy check-out

Quick and easy check-out

You can quickly pay through credit card or Stuvia-credit for the summaries. There is no membership needed.

Focus on what matters

Focus on what matters

Your fellow students write the study notes themselves, which is why the documents are always reliable and up-to-date. This ensures you quickly get to the core!

Frequently asked questions

What do I get when I buy this document?

You get a PDF, available immediately after your purchase. The purchased document is accessible anytime, anywhere and indefinitely through your profile.

Satisfaction guarantee: how does it work?

Our satisfaction guarantee ensures that you always find a study document that suits you well. You fill out a form, and our customer service team takes care of the rest.

Who am I buying these notes from?

Stuvia is a marketplace, so you are not buying this document from us, but from seller FullyFocus. Stuvia facilitates payment to the seller.

Will I be stuck with a subscription?

No, you only buy these notes for $11.33. You're not tied to anything after your purchase.

Can Stuvia be trusted?

4.6 stars on Google & Trustpilot (+1000 reviews)

73091 documents were sold in the last 30 days

Founded in 2010, the go-to place to buy study notes for 14 years now

Start selling
$11.33
  • (0)
  Add to cart